Physics Video Lecture
Connecting Students in 221 Countries

Video Lectures

High School/JEE/NEET/IPhO Physics | 17-19 Yrs

Motion in One Dimension
Module - 1 Introduction
Duration: 13.56 Min
 
Add to my sequences
Bookmark This Video
Duration: 5.23 Min
 
Add to my sequences
Bookmark This Video
Duration: 2.26 Min
 
Add to my sequences
Bookmark This Video
Duration: 2.33 Min
 
Add to my sequences
Bookmark This Video
Module - 5 Acceleration
Duration: 10.41 Min
 
Add to my sequences
Bookmark This Video
Duration: 3.38 Min
 
Add to my sequences
Bookmark This Video
Duration: 3.51 Min
 
Add to my sequences
Bookmark This Video
Duration: 4.12 Min
 
Add to my sequences
Bookmark This Video
Duration: 11.18 Min
 
Add to my sequences
Bookmark This Video
Module - 10 Solved Example-6
Duration: 3.11 Min
 
Add to my sequences
Bookmark This Video
Module - 11 Solved Example-7
Duration: 4.41 Min
 
Add to my sequences
Bookmark This Video
Module - 12 Solved Example-8
Duration: 2.29 Min
 
Add to my sequences
Bookmark This Video
Module - 13 Solved Example-9
Duration: 2.24 Min
 
Add to my sequences
Bookmark This Video
Module - 14 Free Fall Motion
Duration: 6.03 Min
 
Add to my sequences
Bookmark This Video
Duration: 2.25 Min
 
Add to my sequences
Bookmark This Video
Duration: 10.24 Min
 
Add to my sequences
Bookmark This Video
Duration: 3.38 Min
 
Add to my sequences
Bookmark This Video
Duration: 5.01 Min
 
Add to my sequences
Bookmark This Video
Duration: 17.11 Min
 
Add to my sequences
Bookmark This Video
Duration: 14.05 Min
 
Add to my sequences
Bookmark This Video
Duration: 13.43 Min
 
Add to my sequences
Bookmark This Video
Duration: 5.25 Min
 
Add to my sequences
Bookmark This Video
  1.  
  2. why i cant just integrate and put x=3 to find distance?
    7 years ago by BHAVESH SAHU

    Post your answer here

  3.  
  4. sir can't i use the limits while finding velocity and then equate to dx/dt? then it is 165m.
    7 years ago by Sudarshana Ghosh

    Post your answer here

  5.  
  6. sir i think sir you have by mistake taken 3^3 as 9 it should be 27 isn't it??
    7 years ago by Samarth Mathur
    Ans 1 ->
    He is right as the equation was 4/3 (t)^3 when you substitute t = 3 we get 4/3 *(3^3) = 27 .
    7 years ago by Vyankatesh Muley
    Ans 2 ->
    sir took 3^3 as 27,but also it was to be divided by 3.so, 27/3 = 9
    4 years ago by

    Post your answer here